LSAT 14 – Section 4 – Question 22

You need a full course to see this video. Enroll now and get started in less than a minute.

Target time: 0:54

This is question data from the 7Sage LSAT Scorer. You can score your LSATs, track your results, and analyze your performance with pretty charts and vital statistics - all with a Free Account ← sign up in less than 10 seconds

Question
QuickView
Type Tags Answer
Choices
Curve Question
Difficulty
Psg/Game/S
Difficulty
Explanation
PT14 S4 Q22
+LR
Weaken +Weak
A
89%
165
B
2%
156
C
3%
156
D
4%
156
E
2%
159
140
148
155
+Medium 148.739 +SubsectionMedium
This page shows a recording of a live class. We're working hard to create our standard, concise explanation videos for the questions in this PrepTest. Thank you for your patience!

This is a weakening question, as the question stem asks: Which one of the following, if true, casts the most doubt on the accuracy of the above conclusion?

This stimulus is placed in quotes without an identified speaker, but this doesn’t really matter beyond allowing some of the referential phrasing such as this company. It begins with the context that the speaker’s company will not be training more pilots, as they have a waitlist of 400 trained pilots. Alright, that kind of makes sense. This decision is then supported by an argument.

The argument begins with the premise that the five other major companies have roughly ~ 400 pilots as well. The stimulus continues with another premise with the support indicator since, stating that each company is going to only need roughly ~100 pilots. From these two premises about the company waitlists and personnel needs, the author concludes that there will be no shortage of personnel. The assumption this argument depends on is that the individuals waitlisted for each company are distinct. Since each company requires about ~100 people and there are 5 companies, around ~500 pilots are needed by the major companies. For the argument to conclude that there are enough trained pilots to avoid a shortage, it must assume that it isn’t the same 400 pilots waitlisted for each company, as in that case there would actually already be a shortage brewing since there are ~100 less pilots than needed. An answer which picks up on this assumption and contradicts it will be an excellent weakening answer. Let’s see what we get:

Correct Answer Choice (A) This does exactly what we identified with our pre-phrase, it directly contradicts the arguments assumption that there isn’t significant overlap between the waitlists.

Answer Choice (B) Our author’s argument is specifically about the forseeable future, the argument’s conclusion is not about whether or not training will be needed in the long run.

Answer Choice (C) Ok? Our author’s conclusion is a prediction that there will be no shortage in the foreseeable future; whether or not there will be an age gap does nothing to weaken the argument.

Answer Choice (D) If anything this strengthens the argument by supporting the accuracy of the author’s claims.

Answer Choice (E) If other companies are training more pilots, that strengthens the author’s prediction that there won’t be a shortage.

Take PrepTest

Review Results

Leave a Reply